Last visit was: 19 Nov 2025, 05:27 It is currently 19 Nov 2025, 05:27
Close
GMAT Club Daily Prep
Thank you for using the timer - this advanced tool can estimate your performance and suggest more practice questions. We have subscribed you to Daily Prep Questions via email.

Customized
for You

we will pick new questions that match your level based on your Timer History

Track
Your Progress

every week, we’ll send you an estimated GMAT score based on your performance

Practice
Pays

we will pick new questions that match your level based on your Timer History
Not interested in getting valuable practice questions and articles delivered to your email? No problem, unsubscribe here.
Close
Request Expert Reply
Confirm Cancel
705-805 Level|   Weaken|         
User avatar
xhym
Joined: 11 Jun 2024
Last visit: 20 Oct 2025
Posts: 65
Own Kudos:
Given Kudos: 7
Location: Canada
Products:
Posts: 65
Kudos: 83
Kudos
Add Kudos
Bookmarks
Bookmark this Post
User avatar
ledzep10
Joined: 13 May 2025
Last visit: 19 Nov 2025
Posts: 60
Own Kudos:
26
 [1]
Given Kudos: 44
Products:
Posts: 60
Kudos: 26
 [1]
1
Kudos
Add Kudos
Bookmarks
Bookmark this Post
User avatar
tgsankar10
Joined: 27 Mar 2024
Last visit: 19 Nov 2025
Posts: 281
Own Kudos:
390
 [1]
Given Kudos: 83
Location: India
Posts: 281
Kudos: 390
 [1]
1
Kudos
Add Kudos
Bookmarks
Bookmark this Post
User avatar
DyutiB
Joined: 21 Jul 2024
Last visit: 19 Nov 2025
Posts: 25
Own Kudos:
16
 [1]
Given Kudos: 3
Products:
Posts: 25
Kudos: 16
 [1]
1
Kudos
Add Kudos
Bookmarks
Bookmark this Post
More gym memberships are bought in the town of Schwartz than in the town of Talsberg. Therefore, the citizens of Schwartz exercise more frequently than do the citizens of Talsberg.

Each of the following, if true, weakens the conclusion above EXCEPT:

A. Schwartz has a larger population than Talsberg does.
B. Most citizens of Talsberg work in Schwartz and have memberships in gyms there.
C. The average price of a gym membership is lower in Schwartz than it is in Talsberg.
D. A large, free community gym open to all is available to the citizens of Talsberg.
E. The average citizen of Schwartz spends less time at the gym per week than does the average citizen of Talsberg.


The argument core => No. of memberships bought in S > No. of memberships bought in T -----> Frequency of exercise in S > Frequency of exercise in T

The wrong answers will weaken the reasoning by challenging the core argument that no. of memberships bought has no relation with frequency of exercise by giving proper reasoning. The no. of memberships bought in S is greater than that of in T for different reasons. We have to find the option that does not - either strengthen or has no impact.

A. Weaken
B. Weaken
C. Weaken
D. Weaken
E. Correct.

Answer (E)
User avatar
Praveena_10
Joined: 01 Jan 2024
Last visit: 18 Nov 2025
Posts: 36
Own Kudos:
36
 [1]
Given Kudos: 21
Location: India
GPA: 7.3
WE:Engineering (Energy)
1
Kudos
Add Kudos
Bookmarks
Bookmark this Post
Optin C strengthens the given statement instead of weakening.
User avatar
bomberjack
Joined: 22 Nov 2023
Last visit: 17 Nov 2025
Posts: 67
Own Kudos:
73
 [1]
Given Kudos: 122
GMAT Focus 1: 635 Q84 V81 DI79
GMAT Focus 2: 675 Q88 V82 DI80
GMAT Focus 3: 715 Q88 V86 DI83
GMAT Focus 3: 715 Q88 V86 DI83
Posts: 67
Kudos: 73
 [1]
1
Kudos
Add Kudos
Bookmarks
Bookmark this Post
Quote:
More gym memberships are bought in the town of Schwartz than in the town of Talsberg. Therefore, the citizens of Schwartz exercise more frequently than do the citizens of Talsberg.

Each of the following, if true, weakens the conclusion above EXCEPT:

A. Schwartz has a larger population than Talsberg does.
B. Most citizens of Talsberg work in Schwartz and have memberships in gyms there.
C. The average price of a gym membership is lower in Schwartz than it is in Talsberg.
D. A large, free community gym open to all is available to the citizens of Talsberg.
E. The average citizen of Schwartz spends less time at the gym per week than does the average citizen of Talsberg.

The conclusion is since more gym memberships are bought therefore Citizens of Schwartz exercise more than citizens of Talsberg.
We need to find the option that does not weaken that means is neutral or strengthens the conclusion.
A : Weakens.
Since population of Schwartz is more, hence more memberships are bought. No relation w.r.t citizens exercising more frequently. Alternate Reasoning found -- hence weakens.
B: Weakens.
Not because the citizens of Schwartz exercise more frequently, but because most citizens of Talsberg work in Schwartz and have memberships in gyms there. Alternate Reasoning found -- hence weakens.
C: Weakens
Average price of gym membership is lowe - Alternate Reasoning found -- hence weakens.
D: Weakens
Free community gym found. Hence less memberships. Alternate Reasoning found -- hence weakens.
E: Time at gym per week data does not aim the conclusion nor give any needed info - Neutral. Hence E is the option.
User avatar
MyGMATbuddy
Joined: 27 Jun 2025
Last visit: 11 Aug 2025
Posts: 46
Own Kudos:
29
 [1]
Given Kudos: 5
Location: India
GMAT 1: 730 Q50 V39
GPA: 9.82
Expert
Expert reply
GMAT 1: 730 Q50 V39
Posts: 46
Kudos: 29
 [1]
1
Kudos
Add Kudos
Bookmarks
Bookmark this Post
Conclusion: Citizens of Schwartz exercise more frequently than those in Talsberg, because more gym memberships are bought in Schwartz.

We are looking for the option that does not weaken this conclusion.

A. Schwartz has a larger population
Weakens. More memberships could just be due to more people.

B. Most Talsberg citizens use gyms in Schwartz
Weakens. The memberships in Schwartz may be used by Talsberg citizens.

C. Gym memberships are cheaper in Schwartz
Weakens. Lower prices may explain more memberships, not more exercise.

D. Talsberg has a free community gym
Weakens. Talsberg citizens may be exercising without paid memberships.

E. Schwartz citizens spend less time at the gym per week
Does not weaken. Less time doesn’t mean less frequent visits. Frequency and duration are different.

Correct answer: E

Pankaj Jindal | GMAT Classic 730 | IIM Bangalore | Helping working professionals
Success story: https://gmatclub.com/forum/how-i-scored ... l#p3588949
User avatar
Elite097
Joined: 20 Apr 2022
Last visit: 08 Oct 2025
Posts: 771
Own Kudos:
Given Kudos: 346
Location: India
GPA: 3.64
Posts: 771
Kudos: 553
Kudos
Add Kudos
Bookmarks
Bookmark this Post
Option A: “Schwartz has a larger population than Talsberg does.”

Explanation:
If Schwartz simply has more people, then more gym memberships don’t necessarily mean each person is exercising more. The higher total could be due to population, not frequency of exercise.

Does it weaken?
✅ Yes. It offers an alternative explanation for the statistic — weakens the link between memberships and actual exercise frequency per citizen.



Option B: “Most citizens of Talsberg work in Schwartz and have memberships in gyms there.”

Explanation:
If Talsberg residents are the ones buying gym memberships in Schwartz, then Schwartz’s high gym membership numbers aren’t actually due to Schwartz citizens.

Does it weaken?
✅ Yes. Undermines the assumption that Schwartz citizens are the ones exercising.



Option C: “The average price of a gym membership is lower in Schwartz than it is in Talsberg.”

Explanation:
Cheap memberships may lead to more people buying them, even if they don’t use them much. So the count of memberships may be inflated.

Does it weaken?
✅ Yes. Breaks the link between more memberships and more actual exercise.



Option D: “A large, free community gym open to all is available to the citizens of Talsberg.”

Explanation:
Talsberg citizens may be exercising without needing to buy memberships at all — so fewer memberships doesn’t mean less exercise.

Does it weaken?
✅ Yes. Undermines the idea that fewer memberships = less exercise.



Option E: “The average citizen of Schwartz spends less time at the gym per week than does the average citizen of Talsberg.”

Explanation:
At first glance, this might seem to weaken — but look closely. The conclusion is about frequency (how often), not duration (how long).
Schwartz citizens could be going to the gym more often, but for shorter periods each time.

Example:
• Schwartz: goes 5 times/week, 20 mins each time.
• Talsberg: goes 2 times/week, 1 hour each time.

In this case, Schwartz citizens exercise more frequently, even though they spend less time overall.

Does it weaken?
❌ No. It talks about time, not frequency. So the conclusion still stands.

Ans E
User avatar
Manu1995
Joined: 30 Aug 2021
Last visit: 11 Nov 2025
Posts: 81
Own Kudos:
Given Kudos: 18
Posts: 81
Kudos: 55
Kudos
Add Kudos
Bookmarks
Bookmark this Post
Goal: Find the option that doesn't weaken the link between "more memberships" and "more frequent exercise by Schwartz citizens."

A. Schwartz has a larger population. This explains why there might be more memberships (just more people), but it doesn't challenge whether those people exercise more frequently. It's a neutral fact or a premise. Does NOT weaken.
B. Talsberg citizens work in Schwartz and join gyms there. Weakens: The memberships aren't necessarily Schwartz citizens.
C. Gyms are cheaper in Schwartz. Weakens: More memberships could be due to price, not frequency of use.
D. Free gym in Talsberg. Weakens: Talsberg citizens could exercise frequently without memberships.
E. Schwartz citizens spend LESS time at the gym. Weakens: Directly contradicts "more frequently."
The only one that doesn't weaken the conclusion is A.
User avatar
31120170423
Joined: 21 Jul 2020
Last visit: 19 Jul 2025
Posts: 33
Own Kudos:
26
 [1]
Given Kudos: 4
Posts: 33
Kudos: 26
 [1]
1
Kudos
Add Kudos
Bookmarks
Bookmark this Post
E. The average citizen of Schwartz spends less time at the gym per week than does the average citizen of Talsberg.

More gym memberships are bought in the town of Schwartz than in the town of Talsberg. Therefore, the citizens of Schwartz exercise more frequently than do the citizens of Talsberg.

Each of the following, if true, weakens the conclusion above EXCEPT:

A. Schwartz has a larger population than Talsberg does. (Weakens because a larger population can indicate lower average gym membership per person)
B. Most citizens of Talsberg work in Schwartz and have memberships in gyms there. (Weakens because this would indicate that the number of memberships bought in schwartz is caused by people from both towns, thus our measure of gym membership will be wrong and conclusion hollow)
C. The average price of a gym membership is lower in Schwartz than it is in Talsberg. (Weakens because it brings another contradicting fact for the conclusion)
D. A large, free community gym open to all is available to the citizens of Talsberg. (Weakens because this would suggest that some of the people of Talsberg don't use the gym for exercise and might be the reason why less people in talsberg buy gym membership, which is contradictory to the conslusion)
E. The average citizen of Schwartz spends less time at the gym per week than does the average citizen of Talsberg. (Correct because this gives a fact that directly supports the conclusion)
User avatar
Aarushi100
Joined: 17 Jul 2024
Last visit: 18 Nov 2025
Posts: 56
Own Kudos:
44
 [1]
Given Kudos: 88
Products:
Posts: 56
Kudos: 44
 [1]
1
Kudos
Add Kudos
Bookmarks
Bookmark this Post
Bunuel
More gym memberships are bought in the town of Schwartz than in the town of Talsberg. Therefore, the citizens of Schwartz exercise more frequently than do the citizens of Talsberg.

Each of the following, if true, weakens the conclusion above EXCEPT:

A. Schwartz has a larger population than Talsberg does.
B. Most citizens of Talsberg work in Schwartz and have memberships in gyms there.
C. The average price of a gym membership is lower in Schwartz than it is in Talsberg.
D. A large, free community gym open to all is available to the citizens of Talsberg.
E. The average citizen of Schwartz spends less time at the gym per week than does the average citizen of Talsberg.


 


This question was provided by Experts' Global
for the GMAT Olympics 2025

Win over $30,000 in prizes such as Courses, Admissions Consulting, and more

 

We have to find the option that either supports the conclusion or is irrelevant to the conclusion.


Let's consider each statement.

A. Schwartz has a larger population than Talsberg does.
This weakens the conclusion. If Schwartz has a larger population, then the gym memberships would be more. Doesn't mean the citizens of Schwartz exercise more than Talsberg. Incorrect.

B. Most citizens of Talsberg work in Schwartz and have memberships in gyms there.
This again weakens the conclusion. It provides an explanation why Talsberg has less number of memberships but still the citizens might be exercising more. Incorrect.

C. The average price of a gym membership is lower in Schwartz than it is in Talsberg.
This only talks about why the number of gym memberships might be higher. Tells us nothing about the citizens of Schwartz exercising more. Hence, this makes the conclusion Irrelevant. Thus, this can be our answer. Correct.

D. A large, free community gym open to all is available to the citizens of Talsberg.
This weakens the conclusion. Explains why the citizens of Talsberg might be exercising more than Schwartz while the gym memberships are less. Incorrect.

E. The average citizen of Schwartz spends less time at the gym per week than does the average citizen of Talsberg.
Weakens the conclusion. The average citizen of Schwartz spends less time at the gym per week than Talsberg, this means Schwartz citizens spend less time exercising. Incorrect.
User avatar
Vishnupriyasundar
Joined: 31 Mar 2025
Last visit: 17 Nov 2025
Posts: 9
Own Kudos:
Given Kudos: 36
Location: India
Products:
Posts: 9
Kudos: 1
Kudos
Add Kudos
Bookmarks
Bookmark this Post
A) Schwartz has a larger population than Talsberg does.
• If Schwartz has more people, then it’s not surprising that it has more gym memberships — but that doesn’t necessarily mean that people in Schwartz exercise more per person.
• This weakens the conclusion — it suggests the difference in gym memberships might be due to population, not actual frequency of exercise.

Weakens



(B) Most citizens of Talsberg work in Schwartz and have memberships in gyms there.
• This explains why Schwartz has more memberships, but those memberships may belong to Talsberg residents!
• So Schwartz’s high gym membership count does not mean Schwartz residents exercise more.

Weakens

C) The average price of a gym membership is lower in Schwartz than it is in Talsberg.-
. This explains why there is more membership is Schwartz and if we consider everyone with membership works out then this strengthens the argument
Does not Weaken

D) A large, free community gym open to all is available to the citizens of Talsberg.
• Talsberg citizens may exercise without memberships, so low membership numbers don’t reflect how much they actually work out.

Weakens



(E) The average citizen of Schwartz spends less time at the gym per week than does the average citizen of Talsberg.
• This suggests Schwartz residents exercise less frequently, even though they own more memberships.

Weakens the conclusion
User avatar
samriddhi1234
Joined: 19 Aug 2022
Last visit: 17 Nov 2025
Posts: 45
Own Kudos:
24
 [1]
Given Kudos: 303
Location: India
Schools: Rotterdam'26
GMAT Focus 1: 625 Q80 V85 DI78
GPA: 9.17
Schools: Rotterdam'26
GMAT Focus 1: 625 Q80 V85 DI78
Posts: 45
Kudos: 24
 [1]
1
Kudos
Add Kudos
Bookmarks
Bookmark this Post
IMO C. If the gym membership prices are cheaper, it could still mean that the citizens of Schwartz exercise more frequently than citizens of Talsberg. In the rest of the options, the conclusion is weakened.
Bunuel
More gym memberships are bought in the town of Schwartz than in the town of Talsberg. Therefore, the citizens of Schwartz exercise more frequently than do the citizens of Talsberg.

Each of the following, if true, weakens the conclusion above EXCEPT:

A. Schwartz has a larger population than Talsberg does.
B. Most citizens of Talsberg work in Schwartz and have memberships in gyms there.
C. The average price of a gym membership is lower in Schwartz than it is in Talsberg.
D. A large, free community gym open to all is available to the citizens of Talsberg.
E. The average citizen of Schwartz spends less time at the gym per week than does the average citizen of Talsberg.


 


This question was provided by Experts' Global
for the GMAT Olympics 2025

Win over $30,000 in prizes such as Courses, Admissions Consulting, and more

 

User avatar
SaKVSF16
Joined: 31 May 2024
Last visit: 18 Nov 2025
Posts: 86
Own Kudos:
79
 [1]
Given Kudos: 41
Products:
Posts: 86
Kudos: 79
 [1]
1
Kudos
Add Kudos
Bookmarks
Bookmark this Post
Argument Analysis
The argument assumes that since more gym memberships are bought in S, people exercise more frequently in S than T.

Classic causal relation implied: exercising more frequently > causes > more gym memberships
There could be many cases where this might be false, like what if people in S buy memberships because of some offers but do not actually exercise? Or what if people in T simply have more open areas where they can run instead, so they don't go to the gym to exercise.

We can see that just buying more gym memberships does not imply more exercise.

Question Statement
The question asks us to pick a statement which doesn't weaken this logic - we can use POE to strike out all choices that do weaken the above logic.

Answer Choice Analysis

A. Schwartz has a larger population than Talsberg does. - weakens. Alternative cause presented. This tells us that there are simply more people in S, so in comparison to T, there would be more people signing up for the gym. They could be exercising just as frequently as those in T.

B. Most citizens of Talsberg work in Schwartz and have memberships in gyms there - weakens. if people of T are being counted in the memberships of S, this shows that the number of memberships in S are not reflective of the actual citizens of S going to the gym.

C. The average price of a gym membership is lower in Schwartz than it is in Talsberg - does not weaken. Number of assumptions need to be further made for this to weaken the conclusion. For example, we would need to assume that people in T cannot afford the higher cost. Also, what if the price only varies by a few dollars, would that be a significant factor to deter the people? We don't know.

D. A large, free community gym open to all is available to the citizens of Talsberg. - weakens. If there is a free gym present in T, people could be exercising here just as frequently, instead of paying for gym memberships. Accounts for the less memberships in T.

E. The average citizen of Schwartz spends less time at the gym per week than does the average citizen of Talsberg. - weakens. If people in S are spending lesser time in the gym, that would directly undermine the conclusion that people in S exercise more frequently than in T.

From the above, C seems to be the only one which does not weaken the argument.
User avatar
shwedash
Joined: 27 Mar 2025
Last visit: 19 Oct 2025
Posts: 6
Own Kudos:
2
 [1]
Given Kudos: 68
Posts: 6
Kudos: 2
 [1]
1
Kudos
Add Kudos
Bookmarks
Bookmark this Post
Option C!
It doesn't directly weaken the argument unlike other options.
Option C tells us that gym memberships are lower in Schwartz than in Talsberg, leading the reader to believe that it can be one of the reasons Schwartz' citizens exercise more frequently than the citizens of Talsberg.
User avatar
cyang8
Joined: 04 Jun 2022
Last visit: 02 Nov 2025
Posts: 12
Own Kudos:
Given Kudos: 111
Posts: 12
Kudos: 3
Kudos
Add Kudos
Bookmarks
Bookmark this Post
My guess is E. The others seem to weaken the conclusion, whereas E does not.
User avatar
Tanabhumi
Joined: 29 Apr 2025
Last visit: 29 Oct 2025
Posts: 21
Own Kudos:
13
 [1]
Given Kudos: 31
Posts: 21
Kudos: 13
 [1]
1
Kudos
Add Kudos
Bookmarks
Bookmark this Post
I chose option C because, based on the question, I needed to determine whether people in S City use the gym more frequently than people in T City—or whether there’s no relevant connection between gym usage in the two cities at all.

Option A was somewhat tempting, but it would only slightly weaken the conclusion rather than directly support it. In contrast, option C points out that the price of gym memberships in S City is lower, which makes it reasonable to expect that people there would use the gym more often than those in T City.
User avatar
Rahilgaur
Joined: 24 Jun 2024
Last visit: 19 Nov 2025
Posts: 104
Own Kudos:
Given Kudos: 45
GMAT Focus 1: 575 Q81 V82 DI72
Products:
GMAT Focus 1: 575 Q81 V82 DI72
Posts: 104
Kudos: 74
Kudos
Add Kudos
Bookmarks
Bookmark this Post
More gym memberships are bought in the town of Schwartz than in the town of Talsberg. -> Gym Membership in S > T


Therefore, the citizens of Schwartz exercise more frequently than do the citizens of Talsberg. -> Assumption = more membership more frequent exercise.


A. Schwartz has a larger population than Talsberg does. - Maybe, it doesn't weakens our conclusion, Schwartz people may or may not be exercising frequently.


B. Most citizens of Talsberg work in Schwartz and have memberships in gyms there. - IF true even though gym memberships are bought in the town of Schwartz but Talsberg citizens are exercising more.
C. The average price of a gym membership is lower in Schwartz than it is in Talsberg. If true Schwartz people are just buying membership even if they are not committed to regular exercise. Doesn't weaken much but in comparison to A option it weakens the conclusion.
D. A large, free community gym open to all is available to the citizens of Talsberg. - If True, in Talsberg people are exercising and the need to buy gym membership for same.
E. The average citizen of Schwartz spends less time at the gym per week than does the average citizen of Talsberg. If true, people who have membership in Talsberg are exercising more and lifting up the average exercising time for entire. Again on comparison with the option A it has more weaking effect on the Conclusion.
User avatar
jkkamau
Joined: 25 May 2020
Last visit: 19 Nov 2025
Posts: 132
Own Kudos:
107
 [1]
Given Kudos: 122
Location: Kenya
Schools: Haas '25
GMAT 1: 730 Q50 V46
GPA: 3.5
Products:
Schools: Haas '25
GMAT 1: 730 Q50 V46
Posts: 132
Kudos: 107
 [1]
1
Kudos
Add Kudos
Bookmarks
Bookmark this Post
If the average price in Schwartz is lower than in Talsberg then that could explain the more purchases hence choice C is more of a strengthener
ANS C
Bunuel
More gym memberships are bought in the town of Schwartz than in the town of Talsberg. Therefore, the citizens of Schwartz exercise more frequently than do the citizens of Talsberg.

Each of the following, if true, weakens the conclusion above EXCEPT:

A. Schwartz has a larger population than Talsberg does.
B. Most citizens of Talsberg work in Schwartz and have memberships in gyms there.
C. The average price of a gym membership is lower in Schwartz than it is in Talsberg.
D. A large, free community gym open to all is available to the citizens of Talsberg.
E. The average citizen of Schwartz spends less time at the gym per week than does the average citizen of Talsberg.


 


This question was provided by Experts' Global
for the GMAT Olympics 2025

Win over $30,000 in prizes such as Courses, Admissions Consulting, and more

 

User avatar
smile2
Joined: 17 Jul 2018
Last visit: 17 Nov 2025
Posts: 59
Own Kudos:
85
 [1]
Given Kudos: 29
Posts: 59
Kudos: 85
 [1]
1
Kudos
Add Kudos
Bookmarks
Bookmark this Post
A. Schwartz having a bigger population could explain why it has more gym memberships—it might just be because there are more people, not because they exercise more often. So, the conclusion about exercise frequency may not hold. This weakens the argument.

B. If most Talsberg residents work in Schwartz and use gyms there, that would explain the higher memberships in Schwartz—but those gym-goers are actually from Talsberg. This weakens the argument.

C. This one’s a bit tricky. If gym memberships are cheaper in Schwartz, that could be why more people sign up—not because they’re more into fitness. It gives another reason for higher memberships, though not directly about how often people work out. This weakens as well.

D. Talsberg having a free gym could explain why they have fewer paid memberships, even if lots of people still exercise. That means people in Talsberg might work out just as often, just not through paid gyms. This weakens the argument.

E. This talks about how long people spend at the gym, not how often. But the original argument is about frequency, not time spent. So this doesn’t really challenge the conclusion. This does not weaken the argument.


I will go with option E.
   1   2   3   4   5   6   7   8   9   
Moderators:
GMAT Club Verbal Expert
7443 posts
GMAT Club Verbal Expert
231 posts
188 posts